AFP Questions 8

Réussis tes devoirs et examens dès maintenant avec Quizwiz!

Which one of the following is the major mechanism of action of metformin (Glucophage)? (check one) A. Stimulation of pancreatic insulin release B. Inhibition of glucose production by the liver C. Inhibition of carbohydrate absorption in the small intestine D. Improved insulin sensitivity of skeletal muscle

B. Inhibition of glucose production by the liver Metformin has multiple mechanisms of action, but its main effect on serum glucose results from inhibition of gluconeogenesis in the liver. Sulfonylureas and meglitinides stimulate insulin release from the pancreas, and thiazolidinediones sensitize peripheral tissues to insulin. Carbohydrate absorption in the small intestine is inhibited by the "-glucosidase inhibitors.

Which one of the following medications is most likely to cause hypokalemia? (check one) A. Albuterol (Proventil, Ventolin) B. Doxazosin (Cardura) C. Erythromycin D. Felodipine (Plendil) E. Lisinopril (Prinivil, Zestril)

β-Agonists activate potassium uptake by the cells. This includes bronchodilators and tocolytic agents. Other agents that can induce hypokalemia include pseudoephedrine and insulin. Diuretics, particularly thiazides, can also cause hypokalemia as a result of the renal loss of potassium.

A 62-year-old male underwent percutaneous coronary intervention and placement of two stents for a myocardial infarction yesterday. He is currently taking simvastatin (Zocor), aspirin, lisinopril (Prinivil, Zestril), and hydrochlorothiazide. His last LDL-cholesterol level was 70 mg/dL and his blood pressure is 130/80 mm Hg.Which one of the following additions to his current regimen would be most appropriate at this time? (check one) A. Amlodipine (Norvasc) B. Diltiazem (Cardizem) C. Verapamil (Calan, Verelan) D. Metoprolol (Lopressor, Toprol-XL) E. No changes

β-Blockers are first-line antihypertensive medications for patients with coronary artery disease (CAD) and have been shown to reduce the risk of death by 23% at 2 years. They should also be given to normotensive patients with CAD if tolerated. Cardioselective (β1) β-blockers such as metoprolol and atenolol are preferred, as they cause fewer adverse effects.

Which one of the following agents used for tocolysis has the unique adverse effect of respiratory depression? (check one) A. Magnesium sulfate B. Ritodrine (Yutopar) C. Terbutaline (Brethine, Bricanyl) D. Indomethacin (Indocin) E. Nifedipine (Adalat, Procardia)

Magnesium sulfate infusions must be carefully monitored because respiratory depression is a potential lethal side effect. Reflexes are usually lost first. Terbutaline and ritodrine have the potential to cause respiratory distress in the form of pulmonary edema. They do not cause respiratory depression. Indomethacin and nifedipine are rarely used tocolytics that do not depress respiration.

A 72-year-old male with a serum creatinine level of 1.8 mg/dL (N 0.6-1.5) requires a contrast dye study. Which one of the following is most effective when given prior to the administration of contrast to reduce the risk for contrast-induced renal failure? (check one) A. N-acetylcysteine B. Mannitol C. Furosemide (Lasix) D. Methylprednisolone sodium succinate (Solu-Medrol) E. Diphenhydramine (Benadryl)

Current methods for reducing the risk of renal failure induced by contrast material include adequate hydration and the use of N-acetylcysteine. Neither mannitol nor furosemide has been shown to prevent contrast-induced renal failure. Corticosteroids and antihistamines are useful for the prevention of idiosyncratic reactions to contrast, but are not helpful in reducing the risk of renal failure.

A 15-year-old African-American male presents to the emergency department with a chief complaint of fever, abdominal pain, nausea, and anorexia. In addition to the usual laboratory evaluation, which one of the following imaging modalities would be most helpful for confirming a diagnosis of appendicitis? (check one) A. Plain flat plate and upright radiographs of the abdomen B. An air contrast barium enema C. Abdominal ultrasonography D. A spiral CT scan of the abdomen E. MRI of the abdomen

D. A spiral CT scan of the abdomen A retrospective review of 650 patients with suspected appendicitis showed a sensitivity of 97% and a specificity of 98% for spiral CT. In patients in whom the clinical diagnosis was uncertain, sensitivity was 92% and specificity was 85%. Two prospective studies comparing ultrasonography with spiral CT have favored spiral CT. Ultrasonography is used in women who are pregnant and women in whom there is a high degree of suspicion of gynecologic disease. Abdominal radiography has low specificity and sensitivity for the diagnosis of acute appendicitis. Air contrast barium enema also has low accuracy. Limitations of MRI include increased cost, decreased availability, and increased examination time compared to CT.

A 16-year-old afebrile, otherwise healthy female presents with a 4-day history of right ear pain. She says she has spent a fair amount of time swimming recently. Traction on the pinna causes pain. The erythema and inflammation is limited to the ear canal but there is too much edema to easily visualize the tympanic membrane.Which one of the following would be the most appropriate treatment? (check one) A. Amoxicillin B. Amoxicillin/clavulanate (Augmentin) C. Amoxicillin/clavulanate plus acetic acid 2% topically D. Ciprofloxacin 0.3%/dexamethasone 0.1% (Ciprodex) topically

D. Ciprofloxacin 0.3%/dexamethasone 0.1% (Ciprodex) topically Acute otitis externa is treated with topical antibiotics. Although no preparation has been shown to be most effective, a fluoroquinolone does not create a risk of ototoxicity if the tympanic membrane is not intact. Topical corticosteroids may hasten symptom reduction. Oral antibiotics are not appropriate unless the infection has spread beyond the ear canal, or if the patient has diabetes mellitus or is immunocompromised.

A patient has HELLP syndrome.Which one of the following would be the most appropriate course of action at this point? (check one) A. Continued monitoring, repeating the 24-hour urine collection, and repeating the laboratory studies tomorrow B. Immediate delivery by cesarean section C. Discharge to home on bed rest, with close follow-up D. Induction of labor with oxytocin (Pitocin) if the cervix is favorable

D. Induction of labor with oxytocin (Pitocin) if the cervix is favorable This patient has hemolysis, elevated liver enzymes, and low platelets (HELLP) syndrome and needs to be delivered. There is no reason to delay delivery in a term pregnancy. HELLP syndrome is a form of severe preeclampsia. If the patient has a favorable cervical examination, labor induction with oxytocin is appropriate. If the cervix is unfavorable, cesarean delivery should be considered to expedite delivery.

Pretibial myxedema is a cutaneous manifestation of? (check one) A. Subclinical diabetes mellitus B. Collagen vascular disease C. Hyperlipidemia, type III D. Ischemia E. Graves disease

Pretibial myxedema is a complication of Graves disease, whether it presents as hypo-or hyperthyroidism. It is a dermopathy that most often occurs in the lower legs and results from increased deposition of mucin due to the endocrine abnormality. Diabetes mellitus can cause necrobiosis lipoidica, a lesion on the lower extremities; hyperlipidemia can cause waxy papules; and collagen vascular and ischemic disease can cause urticaria and/or ulceration.

Which one of the following cardiac rhythm abnormalities is most common in patients with anorexia nervosa? (check one) A. Atrial fibrillation B. Ventricular fibrillation C. Sinus bradycardia D. Sinus tachycardia E. Paroxysmal supraventricular tachycardia

Sinus bradycardia is almost universally present in patients with anorexia nervosa. It is hypothesized that this is due to vagal hyperactivity resulting from an attempt to decrease the amount of cardiac work by reducing cardiac output. It is also possible that the bradycardia can be accounted for by low serum T3 levels, a common finding in persons with chronic malnutrition. Sinus tachycardia may occur with refeeding in patients with anorexia. Other arrhythmias may also occur but are less frequent.

For which type of renal calculus is acidification of the urine indicated? (check one) A. Cystine B. Uric acid C C. Calcium oxalate D. Calcium phosphate

Urine pH is an important factor in the production of kidney stones. Uric acid, cystine, and calcium oxalate stones tend to form in acidic urine, whereas struvite (magnesium ammonium phosphate) and calcium phosphate stones form in alkaline urine. Urine should be acidified for prevention of calcium phosphate and struvite stones. Cranberry juice or betaine can lower urine pH.

A 27-year-old male requests your advice regarding colon cancer screening. His brother died of colon cancer, which was diagnosed at the age of 40.You suggest that he begin colonoscopy screening? (check one) A. Now B. At age 30 C. At age 40 D. At age 45 E. At age 50

30 Average-risk adults should be screened for colon cancer starting at 50 years of age, and high-risk adults either at age 40 or 10 years before the age at which colorectal cancer was diagnosed in the youngest affected relative.

Compared to anesthesia using only parenteral opioids, the use of epidural anesthesia in labor and delivery increases the rate of which one of the following? (check one) A. Cesarean section B. Low Apgar scores (<7) C. Maternal low backache 3 months post delivery D. Prolonged second stage of labor

Multiple systematic reviews have been conducted to examine the effects of epidural anesthesia on maternal and neonatal outcomes. There are many confounding variables in the studies and, as a result, only a few effects of epidural anesthesia are consistently seen on a statistically significant basis: an increased duration of the second stage of labor, an increased rate of instrument-assisted vaginal deliveries, and an increased likelihood of maternal fever. Overall, there is no statistically significant difference in the duration of the first stage of labor, the incidence of low Apgar scores, or the incidence of maternal backache at 3 months or 12 months.

A 19-year-old white male with a history of fever, fatigue, weight loss, and mild diarrhea of 2 months' duration is found to have a palpable mass in the right lower quadrant of the abdomen. The most likely diagnosis is: (check one) A. Crohn's disease (regional enteritis) B. Ulcerative colitis C. Amebic colitis D. Diverticulitis E. Lymphoma

When Crohn's disease affects primarily the distal small intestine (regional enteritis), a most characteristic clinical pattern emerges. A young person, usually in the second or third decade, will present with a period of episodic abdominal pain, largely postprandial and often periumbilical, occasionally with low-grade fever and mild diarrhea. Anorexia, nausea, and vomiting may also be present. Weight loss is frequent. Some patients may be aware of tenderness in the right lower quadrant and even of a palpable mass in that region.

A 56-year-old female presents for a health maintenance examination. She has a history of a total hysterectomy for benign disease 4 years ago. You are able to document that the hysterectomy pathology was benign and that she has had normal Papanicolaou (Pap) tests for 10 years. The patient asks about regular Pap smears. Which one of the following would be the most appropriate recommendation? (check one) A. Routine Pap smears should be continued until age 70 B. A Pap smear should be done every 3 years C. A Pap smear is not indicated D. A Pap smear should be done yearly for 3 years and only if indicated thereafter

After a hysterectomy for documented benign disease, cytologic screening may be discontinued. Papanicolaou (Pap) smears in this population are low yield and may cause unnecessary testing due to false-positives. Pap smears may be continued if the reason for the hysterectomy is uncertain. If there is a history of invasive cervical cancer or DES exposure, screening should be continued, although there is not a great deal of data to support this practice.

A 59-year-old female with type 2 diabetes develops a 2x1-cm ulcer on the plantar aspect of her right foot. The ulcer is very deep and there is surrounding cellulitis. A plain film is normal. Which one of the following would be the imaging study of choice to rule out osteomyelitis in this patient? (check one) A. Angiography B. A CT scan C. An MRI scan D. A PET scan E. A leukocyte scan

Although leukocyte scans are sensitive for the diagnosis of foot ulcers, MRI is now considered the imaging study of choice when osteomyelitis is suspected; the sensitivity and specificity of MRI in diabetic patients are 90% or greater.

Which one of the following is most consistent with a diagnosis of asthma? (check one) A. Reduced FEV1 and a decreased FEV1/FVC ratio B. Reduced FEV1 and a normal FEV1/FVC ratio C. Reduced FEV1 and an increased FEV1/FVC ratio D. Reduced FVC and a normal FEV1/FVC ratio E. Reduced FVC and an increased FEV1/FVC ratio

A. Reduced FEV1 and a decreased FEV1/FVC ratio Asthma is typically associated with an obstructive impairment that is reversible with short-acting bronchodilators. A reduced FEV1 and a decreased FEV1/FVC ratio indicates airflow obstruction. A reduced FVC with a normal or increased FEV1/FVC ratio is consistent with a restrictive pattern of lung function.

An 11-year-old male is brought to your office for evaluation of bilateral posterior heel pain that has occurred for the past few months. He plays basketball and soccer several times a week and the pain begins several minutes into each of these activities. There is no pain at rest or with walking. The patient has not noticed any numbness, tingling, or weakness.On examination you find no swelling or tenderness of the heel or Achilles tendon. Reflexes, strength, and range of motion at the ankle are intact, but he does have bilateral posterior heel pain when you passively dorsiflex the ankles.Which one of the following is the most likely diagnosis? (check one) A. Achilles tendinopathy B. Calcaneal apophysitis C. Plantar fasciitis D. Heel pad syndrome E. Tarsal tunnel syndrome

Calcaneal apophysitis, also known as Sever disease, is the most common etiology of heel pain in children,usually occurring between 5 and 11 years of age. It is thought that in these children the bones grow faster than the muscles and tendons. A tight Achilles tendon then pulls on its insertion site at the posterior calcaneus with repetitive running or jumping activities, causing microtrauma to the area. There may be swelling and tenderness in this area and passive dorsiflexion may increase the pain. Radiography is usuallynormal and therefore does not often aid in the diagnosis, but it may reveal a fragmented or scleroticcalcaneal apophysis. Treatment involves decreasing pain-inducing activities, anti-inflammatory or analgesicmedication if needed, ice, stretching and strengthening of the gastrocnemius-soleus complex, and the useof orthotic devices. Plantar fasciitis and heel pad syndrome cause pain on the plantar surface of the heel rather than posteriorly.Achilles tendinopathy causes tenderness to palpation of the Achilles tendon. Tarsal tunnel syndrome relatedto compression of the posterior tibial nerve causes neuropathic pain and numbness in the posteromedialankle and heel.

Which one of the following statements is true about celiac disease (gluten-sensitive enteropathy) in adults? (check one) A. It is more common among African-Americans B. Symptoms are limited to gastrointestinal complaints C. Type 2 diabetics are at increased risk for the disease D. Serum antibody tests are sensitive and specific E. Colonoscopy with mucosal biopsy is required to make the diagnosis

Celiac disease is thought to be greatly underdiagnosed in the United States. Antibody tests indicate that the prevalence is approximately 1:250 among adult Americans of European ancestry. Approximately 7% of type 1 diabetics have celiac disease. A number of other autoimmune syndromes have been associated with celiac disease, including thyroid disease and rheumatoid arthritis. There is no reported association with type 2 diabetes. Gastrointestinal involvement may manifest as diarrhea, constipation, or other symptoms of malabsorption, such as bloating, flatus, or belching. Fatigue, depression, fibromyalgia-like symptoms, aphthous stomatitis, bone pain, dyspepsia, gastroesophageal reflux, and other nonspecific symptoms may be present and can make the diagnosis quite challenging. Dermatitis herpetiformis is seen in 10% of patients with celiac disease. Serum antibody testing, especially IgA antiendomysial antibody, is highly sensitive and specific and readily available at a cost of about $100 to $200. Definitive diagnosis generally requires esophagogastroduodenoscopy with a biopsy of the distal duodenum to detect characteristic villous flattening.

Total parenteral nutrition is most appropriate for patients: (check one) A. With poorly functioning gastrointestinal tracts who cannot tolerate enteral feeding B. Who cannot swallow because of an esophageal motility problem C. Who refuse to eat D. In whom maintenance nutrition is desired for a short period following recovery from surgery

Total parenteral nutrition (TPN) is indicated for patients with poorly functioning gastrointestinal tracts who cannot tolerate other means of nutritional support and for those with high caloric requirements that cannot otherwise be met. Patients who cannot swallow because of an esophageal motility problem and those who are resistant to feeding can be managed with tube feedings. Peripheral alimentation, which provides fewer calories than TPN or liquid tube feedings, would be more appropriate over the short term in patients recovering from surgery.

A 58-year-old male who works with heavy machinery at a local factory presents to your office for evaluation of hearing loss of several years' progression. He notes that the loss is mainly in the left ear and he also has mild tinnitus. He has had no trauma to his head, and he has no history of ear infections. Examination of the ears reveals normal tympanic membranes and a neurologic examination is negative. When a tuning fork is placed in the center of his forehead, he says the sound is much louder on the right side (Weber test). Comparing sound in front of the ear to the sound when the tuning fork is placed on the mastoid (the Rinne test) reveals that air conduction is better than bone conduction in the left ear. Which one of the following is true regarding further evaluation and management? (check one) A. No treatment or further diagnostic studies are indicated B. A hearing aid plus better hearing protection is all that is needed C. Carotid ultrasonography should be ordered D. A tympanogram is indicated E. Audiometry is the best initial screening test

E. Audiometry is the best initial screening test Acoustic neuroma symptoms are due to cranial nerve involvement and progression of tumor size. Hearing loss is present 95% of the time and tinnitus is very common. The loss is usually chronic (over 3 years) and as many as one-third of patients are unaware it has occurred. Vestibular nerve involvement most often causes mild unsteadiness and rarely has accompanying true vertigo. Trigeminal involvement can cause pain, paresthesias, or numbness of the face. Facial paresis occurs 6% of the time. The diagnosis of acoustic neuroma is based on asymmetric sensorineural hearing loss or another cranial nerve deficit, with confirmation based on MRI with gadolinium contrast or a CT scan. The best initial screening laboratory test is audiometry, as only 5% of patients with acoustic neuroma will have a normal test. Sensorineural loss is usually in the higher frequencies. Brainstem-evoked response audiometry may be used as a further screening measure when there are unexplained symmetrics and standard audiometric testing.

Which one of the following is a contraindication to the use of combined hormonal contraceptives? (check one) A. A family history of breast cancer in a first degree relative B. Rheumatoid arthritis treated with immunosuppression C. Morbid obesity D. Migraine headaches with aura E. Ovarian cancer

D. Migraine headaches with aura The U.S. Medical Eligibility Criteria for Contraceptive Use were created to guide health care providers in assessing the safety of contraceptive use for patients with specific conditions. Category 1 includes conditions for which no restrictions exist for use of the contraceptive method. Category 2 indicates that the method generally can be used, but careful follow-up may be required. Category 3 is used to classify conditions for which the method usually is not recommended unless more-preferred methods are not available or acceptable. Category 4 comprises conditions that represent an unacceptable health risk if the method is used. For combined hormonal contraceptives, migraine headaches with aura at any age are classified as category 4 because of the increased risk of ischemic stroke.

Activated protein C resistance (factor V Leiden) is most commonly found in patients with? (check one) A. Hemolytic anemia B. Carcinoma of the lung C. Familial hypercholesterolemia D. Venous thrombotic disease E. Cystic fibrosis

D. Venous thrombotic disease Venous thrombosis, both acute and recurrent, is associated with several hematologic abnormalities, in addition to the well-known factors of trauma, surgery, malignancy, sepsis, and oral contraceptive use. Notably, activated protein C resistance (factor V Leiden) has been found to be one of the most common hereditary causes of thrombophilia.

A 47-year-old postmenopausal female falls while carrying groceries into her house and sustains a right distal radial fracture. A chemistry panel reveals a calcium level of 11.2 mg/dL (N 8.6-10.6) and further evaluation leads to a diagnosis of primary hyperparathyroidism.Which one of the following is the best course of treatment for this patient? (check one) A. Estrogen replacement therapy B. Long-term bisphosphonate therapy C. Daily furosemide treatment with increased oral fluids D. Elimination of calcium and vitamin D from the diet E. Referral to a surgeon for consideration of parathyroidectomy

E. Referral to a surgeon for consideration of parathyroidectomy Hyperparathyroidism is usually caused by a single adenoma of one of the four parathyroid glands. A minority of cases (10%-15%) are associated with four-gland hyperplasia. Studies that localize the glands, such as a technetium scan or ultrasonography, help surgeons who are familiar with this condition achieve a cure rate of 95%-98%, with an estimated complication rate of 1%-3%. For patients <50 years old or symptomatic patients, such as those with a fragility fracture, parathyroidectomy is the treatment of choice. If a patient is older, is a poor surgical candidate, or has asymptomatic disease, long-term monitoring with treatment focused on reducing bony complications can be considered (SOR C).

An 80-year-old male presents with the chief complaint of a "bone spur," describing mid-heel pain that worsens as the day progresses. The pain is not relieved with ibuprofen. Examination reveals tenderness in the central aspect of the heel and a radiograph of the foot is unremarkable. The most likely diagnosis is: (check one) A. Multiple myeloma B. Fat-pad atrophy C. Tarsal tunnel syndrome D. S1 radiculopathy E. Plantar fasciitis

Fat-pad atrophy is a common cause of heel pain in the geriatric patient, and in contrast to plantar fasciitis, causes pain as the day progresses. Plantar fasciitis classically presents as morning pain. Tarsal tunnel syndrome causes neuropathic pain in the distribution of the posterior tibial nerve, radiating into the plantar aspect of the foot toward the toes. Lumbar radiculopathy involves pain radiating down the leg into the heel, and is usually associated with weakness of dorsiflexion of the big toe and a decreased ankle reflex. Multiple myeloma would be an extremely unusual cause of heel pain; heel pain associated with cancer more commonly presents nocturnally.

A 45-year-old male presents with a 3-month history of hoarseness. He denies any other complaints and has not been ill recently. He is not on any medication, has no history of chronic medical problems, and does not smoke cigarettes or drink alcohol.Which one of the following would be the most appropriate management of this patient? (check one) A. Voice rest for 1 month B. Laryngoscopy C. A trial of a proton pump inhibitor D. A trial of inhaled corticosteroids E. Oral corticosteroids

Laryngoscopy should be performed to visualize the larynx and evaluate for vocal cord pathology in a patient whose hoarseness does not resolve within 3 months (SOR C). If a serious condition is suspected for some other reason, laryngoscopy should be performed regardless of the duration of symptoms. If there is a recent history of upper respiratory infection or vocal abuse, then it would be appropriate to recommend voice rest for 2 weeks. Laryngoscopy would then be indicated if the hoarseness did not improve or recurred after voice rest. For patients with symptoms of gastroesophageal reflux, a trial of a proton pump inhibitor is recommended (SOR B). Inhaled corticosteroids, especially fluticasone, may cause hoarseness. Oral corticosteroids do not have a role in the management of hoarseness.

Which one of the following is the most effective initial treatment of head lice in an 8-year-old child? (check one) A. Lindane (Kwell) B. Wet combing every 4 days, to continue for 2 weeks after any louse is found C. Head shaving D. Nightly application of petrolatum to the scalp, covered by a shower cap E. Malathion (Ovide)

Malathion is currently the most effective treatment for head lice and is less toxic than lindane. Permethrin and pyrethrins are less effective than malathion, although they are acceptable alternatives. These insecticides, as well as lindane, are not recommended in children 2 years of age or younger. Wet combing may be effective, but is less than half as effective as malathion. Head shaving is only temporarily effective and is traumatic. Petrolatum is not proven to be effective.

Which one of the following medications is most appropriate for treating moderate to severe shortness of breath in a hospice patient with lung cancer? (check one) A. Dexamethasone B. Haloperidol C. Scopolamine D. Morphine

Morphine effectively decreases the feeling of shortness of breath in hospice patients. Randomized, controlled trials have shown significant improvements in symptoms without a significant change in oxygen saturation. Haloperidol can be used for nausea and vomiting (SOR B) and delirium, but is not helpful in the treatment of shortness of breath. Scopolamine is used to decrease the production of secretions but is not helpful for treating dyspnea. Corticosteroids will not manage the sensation of shortness of breath in a dying patient.

A 5-year-old African-American male presents with behavior problems noted in the first 3 months of kindergarten. The mother explains that the child does not pay attention and often naps in class. He averages 10 hours of sleep nightly and is heard snoring frequently. The mother has a history of attention-deficit disorder and takes atomoxetine (Strattera). The boy's examination is within normal limits except for his being in the 25th percentile for weight and having 3+ tonsillar enlargement. The most reasonable plan at this point would include which one of the following? (check one) A. An electroencephalogram B. Polysomnography C. Atomoxetine D. Methylphenidate (Ritalin)

Obstructive sleep apnea is increasingly recognized in children. The peak incidence is in the preschool-age range of 2-5 years when adenotonsillar tissue is greatest in relation to airway size. It is associated with obesity in older children. Common clinical manifestations include snoring with sleep interruptions and respiratory pauses. Polysomnography is the gold standard for the diagnosis. Although the child has inattention, excessive drowsiness is not seen in attention-deficit/hyperactivity disorder (ADHD) and medications for that condition are not indicated. None of his symptoms suggests a seizure disorder, so an EEG would not be helpful.

The most common cause of proteinuria in children is: (check one) A. Acute postinfectious glomerulonephritis B. Lupus glomerulonephritis C. Hydronephrosis D. Orthostatic proteinuria E. Reflux nephropathy

Orthostatic proteinuria accounts for up to 60% of all cases of asymptomatic proteinuria reported in children, with an even higher incidence in adolescents.

A 25-year-old primigravida asks about pain management during labor. You inform her that use of regional analgesia during labor? (check one) A. Increases the likelihood of cesarean delivery B. Increases the risk for instrument-assisted vaginal delivery C. Provides less pain relief than opioid analgesia D. Lowers 1-minute Apgar scores

Regional analgesia in laboring patients increases the risk of vacuum-or forceps-assisted delivery (relative risk = 1.42; 95% confidence interval, 1.28-1.57; 23 trials; n = 735). Multiple randomized, controlled trials have compared regional analgesia with no analgesia. In a meta-analysis, no statistically significant impact was found on the risk of cesarean delivery, maternal satisfaction with pain relief, long-term backache, or immediate effect on neonatal status as determined by Apgar scores. Regional analgesia provides better pain relief than opioid analgesia.

A patient with ascites is suspected to have secondary hyperaldosteronism. Which one of the following would be typical levels of electrolytes in an aliquot specimen of urine? (check one) A. Sodium 2 mEq/L, potassium 40 mEq/L B. Sodium 5 mEq/L, potassium 0 mEq/L C. Sodium 40 mEq/L, potassium 40 mEq/L D. Sodium 80 mEq/L, potassium 2 mEq/L E. Sodium 100 mEq/L, potassium 20 mEq/L

Secondary hyperaldosteronism is characterized by sodium retention, and thus decreased urinary sodium excretion, while potassium secretion is normal to increased.

The most common stress fracture in children involves which one of the following bones? (check one) A. Calcaneus B. Tibia C. Fibula D. Tarsal navicular E. Metatarsal

Tibial fractures are the most common lower extremity stress fractures in both children and adults, accounting for about half of all stress fractures.

Metformin (Glucophage), which is normally used in the management of diabetes mellitus, has also been shown to have a beneficial effect in: (check one) A. Osteoporosis B. Hyperthyroidism C. Polycystic ovary syndrome D. Right ventricular hypertrophy E. Morbid truncal obesity

Recent data suggest that insulin resistance and hyperinsulinemia are important in the pathogenesis of polycystic ovary syndrome (POS). Treatment with drugs that reduce insulin levels, such as metformin, has been shown to correct many of the metabolic abnormalities associated with POS. Such correction results in resumption of ovulation, decreased insulin resistance, and improved beta-cell function; it also produces improvement in cardiovascular risk factors such as dyslipidemia and impaired fibrinolysis.

A 26-year-old gravida 2 para 1 at 10 weeks' gestation presents to the emergency department with abdominal pain and vaginal spotting. Ultrasonography reveals an ectopic pregnancy. Her blood type is A-negative, antibody-negative. Appropriate management with regard to her Rh status includes: (check one) A. Administration of 50 µg of RHO immune globulin (RhoGAM) B. Administration of 300 µg of RhoGAM C. Administration of 50 µg of RhoGAM only if she requires laparoscopic intervention D. Administration of 300 µg of RhoGAM only if she requires laparoscopic intervention E. No RhoGAM, as it is not indicated in an Rh-negative woman with an ectopic pregnancy

A. Administration of 50 µg of RHO immune globulin (RhoGAM) Both ectopic pregnancy and spontaneous or therapeutic abortion pose a significant risk for fetomaternal hemorrhage. Thus, administration of RHO immune globulin (RhoGAM) is recommended in any Rh-negative patient who is unsensitized (D antibody screen-negative prior to administration of RhoGAM). If the estimated gestational age is 12 weeks or less, 50 mcg of RhoGAM is recommended. If the estimated gestational age is greater than 12 weeks, 300 µg of RhoGAM is recommended.

A 22-year-old primigravida presents for routine prenatal care at 18 weeks gestation. She is frustrated because of increased pigmentation on her face consistent with melasma (chloasma).Which one of the following would you recommend for this patient? (check one) A. Use of a high-potency broad-spectrum sunscreen B. Use of hydroquinone for 4 weeks C. Postpartum use of oral contraceptives D. Avoiding future use of topical retinoids E. Increased surveillance for skin cancer beginning at age 40

A. Use of a high-potency broad-spectrum sunscreen Melasma is a very common condition in pregnancy and is due to hyperpigmentation related to normal hormonal changes that accompany pregnancy. It can also be caused by oral contraceptives and is more common in dark-skinned persons.High-potency broad-spectrum sunscreens may help prevent melasma, or at least prevent worsening of the condition (SOR C). Topical retinoids, hydroquinone, and corticosteroids can also be helpful, but are usually reserved for postpartum use and require months of treatment. Other treatments include azelaic acid, chemical peels, kojic acid, cryosurgery, and laser treatment (SOR B). Melasma usually improves spontaneously after delivery, but it may be prolonged or worsened by oral contraceptive use. It does not increase the risk of developing skin malignancies.

A 72-year-old white male has new-onset hypertension with a current blood pressure of 190/110 mm Hg. Which one of the following agents can be used as part of a test for diagnosing renovascular hypertension, but would also increase the risk for azotemia if used for treatment? (check one) A. Captopril (Capoten) B. Metoprolol (Lopressor) C. Clonidine (Catapres) D. Furosemide (Lasix) E. Amlodipine (Norvasc)

ACE inhibitors can significantly worsen renal failure in patients with hypertension caused by renovascular disease. Hyperkalemia is an associated problem. Captopril renography is a useful diagnostic screening test. The other agents are useful for lowering blood pressure but may cause mild creatinine elevations. They do not, however, cause the significant elevations of creatinine seen with ACE inhibitors in cases of significant renovascular disease.

A 45-year-old white male presents with severe intermittent right flank pain that radiates into his right groin area. You suspect a ureteral stone. Which one of the following would most reliably confirm your suspected diagnosis? (check one) A. A helical CT scan of the abdomen and pelvis without contrast B. Intravenous pyelography C. Abdominal ultrasonography D. A KUB plain film of the abdomen E. A urinalysis

An unenhanced helical CT scan of the abdomen and pelvis is the best study for confirming the diagnosis of a urinary tract stone in a patient with acute flank pain, supplanting the former gold standard, intravenous pyelography. A CT scan may also reveal other pathology, such as appendicitis, diverticulitis, or abdominal aortic aneurysm. Although abdominal ultrasonography has a very high specificity, it is still not better than CT, and its sensitivity is much lower; thus, its use is usually confined to pregnant patients with a suspected stone. Plain abdominal radiographs may show the stone if it is radiopaque, and are useful for following patients with radiopaque stones. CT will reveal a radiopaque stone. While most patients with stones will have hematuria, its absence does not rule out a stone.

A 35-year-old nulligravida sees you for preconception counseling. She has hypothyroidism treated with levothyroxine (Synthroid), and her most recent TSH level was in the therapeutic range. She has no symptoms of hypothyroidism.Which one of the following is the patient most likely to require if she becomes pregnant? (check one) A. A decreased dosage of levothyroxine B. An increased dosage of levothyroxine C. The addition of liothyronine (Cytomel) D. Substitution of desiccated thyroid hormone preparation (Armour Thyroid) for the levothyroxine

B Thyroid hormone requirements increase during pregnancy. Most women with hypothyroidism who become pregnant require an increased levothyroxine dosage (SOR A). A common recommendation is to have women on fixed daily doses of levothyroxine begin taking nine doses weekly (one extra dose on 2 days of the week) as soon as the pregnancy is confirmed (SOR B). Thyroid function tests should be repeated regularly throughout the pregnancy to guide additional dosage adjustments.

A 72-year-old white female who is otherwise healthy complains of occasional incontinence. She reports that this occurs mainly at night when she awakens with an intense desire to void, and by the time she is able to get to the bathroom she has "wet herself." The most likely diagnosis is: (check one) A. Sphincter incompetence B. Detrusor instability C. Detrusor hypotonia D. Uninhibited neurogenic bladder

B. Detrusor instability Urinary incontinence is very common in the elderly female. Treatment depends entirely on a careful history to ascertain the exact circumstances when the patient wets herself. One of the most common types of incontinence results from uninhibited contractions of the detrusor muscle. This detrusor instability causes an intense urge to void, which overcomes the patient's voluntary attempt to hold the sphincter closed; hence, the common term urge incontinence. Other common causes of incontinence include a weak sphincter (sphincter incompetence), which leads to leakage associated with ordinary activities such as coughing or lifting (stress incontinence). Another common cause is overflow of urine from an abnormally distended, hypotonic, poorly contractile bladder (detrusor hypotonia). This is probably more common in males with longstanding obstruction due to prostatic hypertrophy. A rare type of incontinence is caused by spinal cord damage. This reflex incontinence is due to the patient being unable to sense the need to void.

The parents of a 40-day-old infant bring her to your clinic because she has had a persistent fever for the past 2 days with rectal temperatures up to 101.0°F. She has been fussy and wants to be held, but has been nursing well. She is crying when you enter the room, and on examination she has good skin turgor and capillary refill. The examination does not reveal any obvious source of infection. By the time you complete the examination the infant is resting quietly in her father's arms.You obtain a CBC and urinalysis. The WBC count is 12,500/mm3 (N 5000-19,500) with an absolute neutrophil count of 8500/mm3 (N 1000-9000). The urinalysis is within normal limits.Which one of the following would be most appropriate at this time? (check one) A. Home care and parental observation only, as long as the temperature remains under 39.0°C (102.2°F) B. Home care and reevaluation in 24 hours C. Oral antibiotics and reevaluation in 24 hours D. A complete sepsis workup, including blood cultures, stool studies, a chest radiograph, and cerebrospinal fluid studies

B. Home care and reevaluation in 24 hours Most children will be evaluated for a febrile illness before 36 months of age, with the majority having a self-limited viral illness. Nontoxic-appearing febrile infants 29-90 days of age who have a negative screening laboratory workup, including a CBC with differential and a normal urinalysis, can be sent home and followed up in 24 hours (SOR B). A second option is to obtain blood cultures and stool studies, or a chest film if indicated by the history or examination, and spinal fluid studies if empiric antibiotics are to be given. This infant's clinical status did not indicate that any of these additional studies should be performed, and empiric antibiotic treatment is not planned.Observation with no follow-up is an appropriate strategy in nontoxic children, but only if the child is 3-36 months of age and the temperature is under 39.0°C (102.2°F) (SOR B). Nontoxic children 3-36 months of age should be reevaluated in 24-48 hours if their temperature is ≥39.0°C. Although a positive response to antipyretics has been considered an indication of a lower risk of serious bacterial infection, there is no correlation between fever reduction and the likelihood of such an infection.Any infant younger than 29 days, and any infant or child with a toxic appearance regardless of age, should undergo a complete sepsis workup and be admitted for observation until culture results are obtained or the source of the fever is found and treated (SOR A).

In an 11-year-old male with dark brown urine and hand and foot edema, which one of the following would be most suggestive of glomerulonephritis? (check one) A. WBC casts in the urine B. RBC casts in the urine C. Eosinophils in the urine D. Positive serum antinuclear antibody levels E. Elevated C3 and C4 complement levels

B. RBC casts in the urine Acute glomerulonephritis (AGN) in children manifests as brown or cola-colored urine, which may be painless or associated with mild flank or abdominal pain. There are many etiologies of AGN but the most common in children are IgA nephropathy (which may directly follow an acute upper respiratory tract infection) and acute poststreptococcal glomerulonephritis following a streptococcal throat or skin infection (usually 7-21 days later). In cases with more severe renal involvement, patients may develop hypertension, edema, and oliguria. RBC casts are the classic finding on urinalysis in a patient with AGN. WBC casts are seen in acute pyelonephritis, often manifested by high fever, and costovertebral angle or flank pain and tenderness. Patients may also appear septic. Positive serum antinuclear antibodies are associated with lupus nephritis. Urine eosinophils are seen in drug-induced tubulointerstitial nephritis. Serum complement levels are reduced, not elevated, in various forms of acute glomerulopathies, including poststreptococcal AGN.

A 52-year-old male with hypertension complains of increased dyspnea for the past 6 months. He reports that he has increased fatigue and dyspnea with normal activities. There is no cough or chest pain. He has a 30-pack-year history of smoking.On examination his blood pressure is 130/85 mm Hg, pulse rate 90 beats/min, respiratory rate 18/min, and O2 saturation 95% on room air. Heart sounds are normal with no murmurs. Auscultation of the lungs reveals bilateral rhonchi.In addition to ordering a chest radiograph, which one of the following should be performed next in the evaluation of this patient's dyspnea? (check one) A. A B-type natriuretic peptide (BNP) level B. A D-dimer level C. Arterial blood gas measurement D. Spirometry E. High-resolution CT of the chest

Based on this patient's history and physical examination, COPD is the most likely cause of his dyspnea. Initial testing should include spirometry to diagnose airflow obstruction (SOR C). CT, a BNP level, a D-dimer level, and arterial blood gas measurements would not be the best initial tests in the evaluation of this patient's dyspnea.

A 40-year-old female is scheduled for a cholecystectomy and you wish to estimate her risk for postoperative bleeding. Which one of the following provides the most sensitive method for identifying her risk? (check one) A. Bleeding time B. Prothrombin time (PT) C. Activated partial thromboplastin time (aPTT) D. Bleeding history

Bleeding time, activated partial thromboplastin time (aPTT), and prothrombin time (PT) are relatively poor predictors of bleeding risk. Studies have shown that baseline coagulation assays do not predict postoperative bleeding in patients undergoing general or vascular surgery who have no history that suggests a bleeding disorder. Obtaining a history for evidence of prior bleeding problems is the most sensitive and accurate method of determining a patient's risk.

You see a 55-year-old female for the first time. She has a 2-year history of chronic daily cough; thick, malodorous sputum; and occasional hemoptysis. She has been treated with antibiotics for recurrent respiratory infections, but is frustrated with her continued symptoms. She has never smoked. Her FEV1/FVC ratio is 60% and CT shows bronchial wall thickening and luminal dilation.The most likely diagnosis is? (check one) A. Emphysema B. Bronchiectasis C. Chronic bronchitis D. Bronchiolitis E. Asthma

Bronchiectasis is an illness of the bronchi and bronchioles involving obstructive and infectious processes that injure airways and cause luminal dilation. In addition to daily viscid, often purulent sputum production with occasional hemoptysis, wheezing and dyspnea occur in 75% of patients. Emphysema and chronic bronchitis, forms of COPD, also cause a decreased FEV1/FVC ratio, but the sputum is generally mucoid and luminal dilation of bronchi is not characteristically present. Bronchiolitis is usually secondary to respiratory syncytial virus infection in young children. Asthma is not characterized by the sputum and CT findings seen in this patient.

A 21-year-old primigravida at 10 weeks gestation has a negative titer for rubella.The best procedure to follow is to? (check one) A. Institute a (-globulin regimen and maintain it throughout her pregnancy B. Administer rubella vaccine after 12 weeks gestation C. Administer rubella vaccine immediately post partum D. Administer rubella vaccine 12 weeks post partum

C. Administer rubella vaccine immediately post partum Rubella has been directly responsible for inestimable pregnancy wastage, as well as for severe congenital malformations. Identification and vaccination of unimmunized women immediately after childbirth or abortion is recommended. The use of (-globulin to prevent viremia in nonimmune subjects exposed to rubella is not recommended. The vaccine should be avoided shortly before or during pregnancy since it is an attenuated live virus. Because of herd immunity there is a very low likelihood that this patient will be exposed to rubella.

A 56-year-old male with diabetes mellitus, hypertension, and chronic renal insufficiency presents for follow-up of his chronic medical conditions. Results of his most recent metabolic panel included an estimated glomerular filtration rate of 30 mL/min/1.73 m2 (N >60) and a calcium level of 10.4 mg/dL (N 8.5-10.2). Medication reconciliation reveals he is not taking the sevelamer (Renagel, Renvela) prescribed by the consulting nephrologist.You explain to the patient that he should be taking sevelamer to lower his serum calcium. The drug accomplishes this by? (check one) A. Blocking the effect of parathyroid hormone B. Blocking excessive vitamin D levels, thus decreasing intestinal calcium absorption and increasing renal calcium excretion C. Blocking intestinal absorption of phosphate, which lowers parathyroid hormone secretion D. Directly blocking excessive calcium absorption in the intestines E. Directly increasing the renal excretion of both calcium and phosphate

C. Blocking intestinal absorption of phosphate, which lowers parathyroid hormone secretion This patient has secondary hyperparathyroidism, a common cause of hypercalcemia in patients with chronic renal insufficiency. Sevelamer is a newer synthetic agent in the therapeutic class of phosphate binders, which includes calcium acetate. Decreasing serum phosphate lowers the feedback stimulation of parathyroid hormone secretion by the parathyroid gland, which is often excessive in chronic renal insufficiency. Normalizing parathyroid levels improves serum calcium levels.

An asymptomatic 32-year-old male requests screening for sexually transmitted diseases. A nucleic acid amplification test is performed on a urine sample, and the results are positive for gonorrhea and negative for Chlamydia. The patient has no known drug allergies.Which one of the following is the recommended treatment for this patient? (check one) A. Ceftriaxone (Rocephin), 125 mg intramuscularly B. Ceftriaxone, 250 mg intramuscularly C. Ceftriaxone, 250 mg intramuscularly, plus azithromycin (Zithromax), 1 g orally D. Ceftriaxone, 125 mg intramuscularly, plus doxycycline, 100 mg orally twice daily for 7 days E. Ciprofloxacin (Cipro), 500 mg orally

C. Ceftriaxone, 250 mg intramuscularly, plus azithromycin (Zithromax), 1 g orally The recommended treatment regimen for gonorrhea is ceftriaxone, 250 mg intramuscularly. The 125-mg regimen is no longer recommended because of treatment failures and limited effectiveness in pharyngeal infections. In addition, the patient should be given azithromycin, 1 g orally, because of the high incidence of coinfection with Chlamydia, even if testing is negative, and to decrease the risk for cephalosporin resistance.

A 56-year-old white male reports lower leg claudication that occurs when he walks approximately one block and is relieved by standing still or sitting. He has a history of diabetes mellitus and hyperlipidemia. His most recent hemoglobin A1c was 5.9% and his LDL-cholesterol level at that time was 95 mg/dL. Current medications include glyburide (DiaBeta), metformin (Glucophage), simvastatin (Zocor), and daily aspirin. He stopped smoking 1 month ago and began a walking program. A physical examination is normal except for barely palpable dorsalis pedis and posterior tibial pulses. Femoral and popliteal pulses are normal. Noninvasive vascular studies of his legs show an ankle-brachial index of 0.7 bilaterally and decreased flow.Which one of the following would be most appropriate for addressing this patient's symptoms? (check one) A. Fish oil B. Warfarin (Coumadin) C. Cilostazol (Pletal) D. Dipyridamole (Persantine) E. Clopidogrel (Plavix)

C. Cilostazol (Pletal) The patient described has symptomatic arterial vascular disease manifested by intermittent claudication. He has already initiated the two most important changes: he has stopped smoking and started a walking program. His LDL-cholesterol is at target levels; further lowering is not likely to improve his symptoms. In the presence of diffuse disease, interventional treatments such as angioplasty or surgery may not be helpful; in addition, these interventions should be reserved as a last resort. Cilostazol has been shown to help with intermittent claudication, but additional antiplatelet agents are not likely to improve his symptoms. Fish oil and warfarin have not been found to be helpful in the management of this conditio

A 42-year-old male with a history of chronic hepatitis C develops left leg cellulitis and is treated with cephalexin (Keflex). He returns to your office 5 days later for follow-up, and the cellulitis is responding favorably to treatment. However, the patient has a generalized maculopapular rash and a low-grade fever, which he says began 3 days ago. He also complains of arthralgias. You admit him to the hospital for further evaluation.His serum creatinine level is 3.2 mg/dL (N 0.6-1.5), which is elevated from his baseline level of 0.8 mg/dL. A urinalysis is normal, except for the presence of occasional eosinophils. The remainder of his evaluation, including liver enzyme levels and renal ultrasonography, is normal.Which one of the following is the most appropriate next step in the management of this patient? (check one) A. A postvoid residual urine volume B. A hepatitis C viral load and genotype C. Discontinuing cephalexin D. Antibiotics to cover methicillin-resistant Staphylococcus aureus (MRSA) E. Aggressive fluid resuscitation with normal saline

C. Discontinuing cephalexin Acute kidney injury (AKI) is currently defined as either a rise in serum creatinine or a reduction in urine output. Creatinine must increase by at least 0.3 mg/dL, or to 50% above baseline within a 24-48 hour period. A reduction in urine output to 0.5 mL/kg/hr for longer than 6 hours also meets the criteria. Acute interstitial nephritis is an intrinsic renal cause of AKI. These patients are often nonoliguric. A history of recent medication use is key to the diagnosis, as cephalosporins and penicillin analogues are the most common causes. Approximately one-third of patients present with a maculopapular rash, fever, and arthralgias. Eosinophilia and sterile pyuria may also be seen in addition to eosinophiluria. Discontinuation of the offending drug is the cornerstone of management. Although up to 30% of patients with chronic hepatitis C infection have some kidney involvement, acute interstitial nephritis is uncommon. Measuring postvoid residual urine volume is indicated if an obstructive cause for the AKI is suspected. Starting an antibiotic to cover methicillin-resistant Staphylococcus aureus (MRSA) is not indicated.

A 53-year-old female is concerned about a skin lesion that has recently been changing in size and shape. On examination she is found to have a 7-mm, asymmetric, darkly pigmented lesion with some color variegation and irregular borders.Which one of the following skin biopsy techniques is most appropriate for confirming the diagnosis? (check one) A. A shave biopsy B. Electrodesiccation and curettage C. Elliiptical excision D. Mohs surgery

C. Elliiptical excision This lesion is suspicious for melanoma, based on the asymmetry, irregular border, color variegation, and size larger than 6 mm. In addition, a history of evolution of the lesion, with changes in size, shape, or color, has been shown in some studies to be the most specific clinical finding for melanoma. The preferred method of biopsy for any lesion suspicious for melanoma is complete elliptical excision with a small margin of normal-appearing skin. The depth of the lesion is crucial to staging and prognosis, so shave biopsies are inadequate. A punch biopsy of the most suspicious-appearing area is appropriate if the location or size of the lesion makes full excision inappropriate or impractical, but a single punch biopsy is unlikely to capture the entire malignant portion in larger lesions. Electrodesiccation and curettage is not an appropriate treatment for melanoma. Mohs surgery is sometimes used to treat melanomas, but is not used for the initial diagnosis.

An 80-year-old female is admitted to your service at a skilled nursing facility 5 days after repair of a hip fracture. When you review her records you note that she has not received any previous treatment for osteoporosis. You are considering ordering zoledronic acid (Reclast) to reduce her risk of another fracture.Which one of the following should be evaluated before administering zoledronic acid to this patient? (check one) A. Vitamin D levels B. Liver enzyme levels C. Estimated glomerular filtration rate D. A CBC

C. Estimated glomerular filtration rate Secondary prevention of fractures is an important component of care following a hip fracture. Options to consider include bisphosphonates, calcium supplementation, and vitamin D supplementation. Bisphosphonates, including zoledronic acid, can reduce rates of clinical fractures among patients who have had a hip fracture (SOR A). While long-term use of bisphosphonates may increase the risk of jaw osteonecrosis and anemia, a CBC is not necessary before initiating therapy. Contraindications to zoledronic acid include hypocalcemia and a creatinine clearance <35 mL/min or other evidence of acute renal impairment.

A 75-year-old white female presents with hyponatremia, with a serum level of 118 mEq/L, a urine osmolality >100 mOsm/kg H2O, and a serum osmolality of 242 mOsm/kg H2O. She complains of some fatigue, but is alert and oriented. Her blood pressure is 136/82 mm Hg. She has normal thyroid, adrenal, cardiac, hepatic, and renal function. You admit her to the hospital for treatment and observation. Which one of the following is the most appropriate initial treatment? (check one) A. Administration of 3% normal saline B. Administration of normal saline C. Free water restriction D. Demeclocycline (Declomycin)

C. Free water restriction This patient probably has the syndrome of inappropriate secretion of antidiuretic hormone (SIADH). SIADH can be caused by CNS tumors, various infections such as meningitis, and pneumonia. Several drugs can cause this condition, including amiodarone, carbamazepine, SSRIs, and chlorpromazine. In this fairly asymptomatic patient, initial management should be free water restriction. As she is hemodynamically stable, she does not need normal saline. Moreover, administration of normal saline may exacerbate the hyponatremia, as the sodium may be rapidly excreted while the water is retained. If she had a rapid onset and neurologic symptoms such as seizures, hypertonic saline could be given. Correction should be slow, with a goal of no more than a 1-2 mmol/L/hr increase in the sodium level; a normal sodium level should not be reached within the first 48 hours of treatment. Demeclocycline is appropriate for patients who cannot adhere to the requirement for fluid restriction, or who have recalcitrant hyponatremia despite restriction.

A 23-year-old gravida 3 para 1 at 28 weeks' gestation whose blood type is O-negative is antibody positive (D antibody) on a routine 28-week screen. Which one of the following best describes the clinical significance of this finding? (check one) A. The fetus HAS hemolytic disease and requires appropriate monitoring and treatment B. The fetus is AT RISK for hemolytic disease only if the biological father is Rh-negative C. The fetus is AT RISK for hemolytic disease only if the biological father is Rh-positive D. The current fetus is NOT at risk for hemolytic disease, but subsequent pregnancies may be at risk

C. The fetus is AT RISK for hemolytic disease only if the biological father is Rh-positive When a person is Rh negative, this indicates that they do not have type D antigen on their red blood cells. If a woman is exposed to Rh D antigen-positive red blood cells, she can have an immune response of variable strength. This may occur in the setting of pregnancy (transplacental fetomaternal transfusion), or exposure outside of pregnancy (e.g., transfusion with mismatched blood). If a maternal antibody screen for D antigen is positive, this indicates that the current fetus MAY be at risk for hemolytic disease. The level of risk is determined by the antibody titer. For example, an antibody titer of 1:4 poses much less risk to the fetus than a titer of 1:64. Determination of the blood type of the father is helpful if paternity is certain. If the father is homozygous Rh negative, there is no risk of alloimmunization to the fetus and the fetus is NOT at risk for hemolytic disease. In this scenario, maternal sensitization occurred either from a prior pregnancy with a different partner or from another source (e.g., transfusion). If the father is heterozygous or homozygous Rh positive, then the fetus IS at risk. If paternity is uncertain, a polymerase chain reaction can be performed on 2 mL of amniotic fluid or 5 mL of chorionic villi to accurately determine the fetal Rh status.

A 70-year-old white female with osteoporosis sees you for follow-up a few days after an emergency department visit for an acute T12 vertebral compression fracture. The fracture was suspected clinically and on plain films; the diagnosis was confirmed with a bone scan. The emergency department physician prescribed oxycodone (OxyContin) and NSAIDs, but the patient is still experiencing considerable discomfort. In addition to increasing the dosage of oxycodone, which one of the following interventions would you suggest now to reduce the patient's pain? (check one) A. Calcitonin (Miacalcin) B. Raloxifene (Evista) C. Alendronate (Fosamax) D. Physical therapy, including dexamethasone iontophoresis E. Vertebroplasty

Calcitonin, either intranasal or subcutaneous, provides pain relief within a few days in many patients with osteoporotic vertebral compression fractures. The remainder of the choices do not provide acute relief. Vertebroplasty/kyphoplasty procedures are generally reserved for cases in which medical management has failed.

You respond to a code blue in the obstetrics department. The patient is a 19-year-old primigravida at 35 weeks gestation, hospitalized with severe preeclampsia. A nurse anesthetist has placed an oral airway and is administering 100% oxygen to the apneic patient. She reports no difficulty ventilating the patient with a bag and valve, and no gagging with oral airway insertion. The patient's blood pressure is 100/60 mm Hg and her pulse rate is 70 beats/min and regular. Her pupils are equal and sluggishly reactive, and she is flaccid and areflexic. The patient had been treated with a magnesium sulfate infusion and a recent bolus of labetalol.Which one of the following medications should you administer initially? (check one) A. Calcium gluconate B. Fosphenytoin C. Labetalol D. Lorazepam (Ativan) E. Dopamine

Calcium gluconate During the treatment of severe preeclampsia with intravenous magnesium, the occurrence of apnea and areflexia is most consistent with magnesium toxicity. In addition to hemodynamic support, calcium infusion is recommended as an antidote. Calcium chloride can be used if a central line has been established. Calcium gluconate would be safer with a peripheral intravenous site.Lorazepam, phenytoin, and fosphenytoin are less useful in preventing eclamptic seizures than magnesium. Labetalol is not indicated given the patient's current blood pressure level. Dopamine, a pressor agent, is not indicated in this scenario, and could aggravate the patient's preeclampsia.

A 4-week-old white male is brought to your office with a 2-week history of increasing dyspnea,cough, and poor feeding. The child appears nontoxic and is afebrile. On examination you noteconjunctivitis, and a chest examination reveals tachypnea and crackles. A chest film showshyperinflation and diffuse interstitial infiltrates and a WBC count reveals eosinophilia.What is the most likely etiologic agent? (check one) A. Staphylococcus species B. Chlamydia trachomatis C. Respiratory syncytial virus D. Parainfluenza virus

Chlamydial pneumonia is usually seen in infants 3-16 weeks of age, and these patients frequently have been sick for several weeks. The infant appears nontoxic and is afebrile, but is tachypneic with a prominentcough. The physical examination will reveal diffuse crackles with few wheezes, and conjunctivitis ispresent in about 50% of cases. A chest film will show hyperinflation and diffuse interstitial or patchyinfiltrates. Staphylococcal pneumonia has a sudden onset. The infant appears very ill and has a fever, and initiallymay have an expiratory wheeze simulating bronchiolitis. Signs of abdominal distress, tachypnea, dyspnea,and localized or diffuse bronchopneumonia or lobar disease may be present. The WBC count will show a prominent leukocytosis. Respiratory syncytial virus infections start with rhinorrhea and pharyngitis, followed in 1-3 days by a cough and wheezing. Auscultation of the lungs will reveal diffuse rhonchi, fine crackles, and wheezes, butthe chest film is often normal. If the illness progresses, coughing and wheezing increase, air hunger andintercostal retractions develop, and evidence of hyperexpansion of the chest is seen. In some infants the course of the illness may be similar to that of pneumonia. Rash or conjunctivitis may occur occasionally,and fever is an inconsistent sign. The WBC count will be normal or elevated, and the differential may benormal or shifted either to the right or left. Chlamydial infections can be differentiated from respiratory syncytial virus infections by a history of conjunctivitis, the subacute onset and absence of fever, and themild wheezing. There may also be eosinophilia.Parainfluenza virus infection presents with typical cold symptoms. Eight percent of infections affect theupper respiratory tract. In children hospitalized for severe respiratory illness, parainfluenza viruses accountfor about 50% of the cases of laryngotracheitis and about 15% each of the cases of bronchitis,bronchiolitis, and pneumonia.

A 14-year-old male has open and closed comedones without evidence of surrounding inflammation on his face and upper back. Which one of the following is the most appropriate initial treatment? (check one) A. Topical antibiotics B. Topical retinoids C. Oral antibiotics D. Oral isotretinoin

Comedones are noninflammatory acne lesions. Inflammatory lesions include papules, pustules, and nodules. Grading acne based on the type of lesion and severity helps guide therapy. Topical retinoids prevent the formation of comedones and reduce their number, and are indicated as monotherapy for noninflammatory acne. Topical antibiotics are used primarily for the treatment of mild to moderate inflammatory or mixed acne. Oral antibiotics are effective for the treatment of moderate to severe acne. Oral isotretinoin is reserved for treatment of severe, recalcitrant acne.

A new drug treatment is shown to reduce the incidence of a complication of a disease by 50%. If the usual incidence of this complication were 1% per year, how many patients with this disease would have to be treated with this medication for 1 year to prevent one occurrence of this complication? (check one) A. 20 B. 50 C. 100 D. 200 E. 500

Considering relative risk reduction without also considering the absolute rate can distort the importance of a therapy. A useful way to assess the importance of a therapy is to determine the number-needed-to-treat for that therapy. To calculate this number, the percentage of absolute risk reduction of a particular therapy is divided into 100. In the case in question, the absolute risk reduction would be 0.5% (0.5x.01). Thus, the number-needed-to-treat for the example cited would be 200 (100/0.5).

Electrosurgical destruction is contraindicated for which one of the following skin lesions? (check one) A. Cherry angiomata B. Pyogenic granuloma C. Basal cell carcinoma D. Melanoma E. Actinic keratosis

Contraindications to treatment with electrosurgery include the use of a pacemaker and the treatment of melanoma. All the other lesions listed can be treated with electrosurgery.

A 50-year-old male presents to your office with erythroderma and fever. He has not had a sore throat, rhinorrhea, cough, or urinary tract symptoms. His current medications include lisinopril (Prinivil, Zestril), atenolol (Tenormin), and allopurinol (Zyloprim). On examination he has a blood pressure of 110/90 mm Hg, a pulse rate of 90 beats/min, and a temperature of 38.6°C (101.5°F). The skin is remarkable for marked erythema over 90% of the body, with tenderness to touch. His mental status is clear and his neck is supple. Mildly tender adenopathy is noted in the neck, axillae, and groin. He has no oral ulcerations or ocular symptoms.A CBC shows a WBC count of 15,000/mm3 (N 4300-10,800) with 20% eosinophils. A metabolic profile shows an AST (SGOT) level of 100 U/L (N 10-40) and an ALT (SGPT) level of 110 U/L (N 10-55), but is otherwise normal.Which one of the following is the most likely diagnosis? (check one) A. Stevens-Johnson syndrome B. Erysipelas C. Red man syndrome D. Toxic shock syndrome E. Drug reaction with eosinophilia and systemic symptoms (DRESS syndrome)

DRESS is an acronym for Drug Reaction with Eosinophilia and Systemic Symptoms. The hallmark of DRESS syndrome is erythroderma accompanied by fever, lymphadenopathy, elevation of liver enzymes, and eosinophilia. The offending medication should be discontinued immediately and treatment with corticosteroids should be initiated. Seizure medications such as carbamazepine, phenytoin, lamotrigine, and phenobarbital are responsible for approximately one-third of cases. Allopurinol-associated DRESS syndrome has the highest mortality rate.Stevens-Johnson syndrome is characterized by a vesiculobullous rash with mucocutaneous involvement, and erysipelas is a painful localized rash with well-demarcated borders. Red man syndrome is associated with vancomycin.

A full-term newborn, born 72 hours ago, is noted to be jaundiced. The pregnancy was uneventful and the delivery uncomplicated. The mother has type A-positive blood and the child has type O-positive. The child is breastfed and has lost 9 ounces from a birth weight of 8 lb. He is feeding for 20 minutes every 4 hours, and except for being icteric, has a normal examination. Laboratory evaluation reveals a total serum bilirubin level of 16 mg/dL (N 1.4-8.7), with a conjugated bilirubin level of 1.0 mg/dL. His hemoglobin level is 17.8 g/dL (N 13.4-19.8), his hematocrit is 55% (N 41-65), and his reticulocyte count is 3% (N 3-7). Appropriate management would include: (check one) A. Phototherapy B. Exchange transfusion C. Blood cultures and antibiotic therapy D. Dextrose and water supplementation E. A recommendation to increase feedings to 10 times a day

E. A recommendation to increase feedings to 10 times a day Hyperbilirubinemia can occur in up to 60% of term newborns during the first week of life. Early guidelines on management of elevated bilirubin were based on studies of bilirubin toxicity in infants who had hemolytic disease. Current recommendations now support the use of less intensive therapy in term newborns with jaundice who are otherwise healthy. Phototherapy should be initiated when the bilirubin level is above 15 mg/dL for infants at age 29-48 hours old, at 18 mg/dL for infants 49-72 hours old, and at 20 mg/dL in infants older than 72 hours. Generally, this problem is not considered pathologic unless it presents during the first hours after birth and the total serum bilirubin rises by more than 5 mg/dL/day or is higher than 17 mg/dL, or if the infant has signs or symptoms suggestive of a serious underlying illness such as sepsis. Fortunately, very few term newborns with jaundice have serious underlying pathology. Physiologic jaundice follows a pattern, with the bilirubin level peaking on the third or fourth day of life and then declining over the first week after birth. Infants with multiple risk factors may develop an exaggerated form of physiologic jaundice, with the total bilirubin level rising as high as 17 mg/dL. Breastfed infants are at increased risk for exaggerated physiologic jaundice because of relative caloric deprivation in the first few days of life. Compared with formula-fed infants, those who are breastfed are six times more likely to experience moderate jaundice, with the bilirubin rising above 12 mg/dL. For breastfed newborns who have an early onset of hyperbilirubinemia, the frequency of feeding should be increased to more than 10 times per day. If the newborn has a decrease in weight gain, delayed stooling, and continued poor intake, then formula supplementation may be necessary. Breastfeeding should be continued to maintain breast milk production. Supplemental water or dextrose and water should not be given, as this can decrease breast milk production and may place the infant at risk for iatrogenic hyponatremia.

Which one of the following is the most appropriate next step in the management of this patient? He has SCFE (slipped capital femoral epiphysis) (check one) A. Reassurance with close follow-up B. Physical therapy C. Injection of the sacroiliac joint D. Hospital admission for tests E. Surgery

E. Surgery The most likely diagnosis for this patient is stable slipped capital femoral epiphysis (SCFE). This is more frequent in males than in females, and is more common in African-Americans and Pacific Islanders than in whites. Although some patients present with pain, many present with a painless limp or vague pain. The average age of onset is 13.5 years for males and 12 years for females. Obesity is strongly associated with SCFE.The lack of systemic symptoms makes osteomyelitis, abscess, or a septic joint much less likely. Malignancy is a possibility, but night pain would be more likely. Sacroiliitis is much less likely given a negative FABER test. The patient's age makes transient synovitis or Legg-Calvé-Perthes disease less likely. Although muscle strain is a possibility, the physical examination findings of external rotation deformity and limited internal rotation are more specific for SCFE.Once the diagnosis of SCFE is made, the patient should not bear weight and should be referred promptly for surgery to prevent complications.

In a 27-year-old white female with irregular menstrual cycles and infertility, which one of the following would be more indicative of Cushing's syndrome rather than the more common polycystic ovarian syndrome? (check one) A. Easy bruising B. Acne C. Hirsutism D. Androgenic alopecia E. Acanthosis nigricans

Easy bruising, moon facies, buffalo hump, abdominal striae, hypertension, and proximal myopathy suggest Cushing's syndrome. Because this syndrome is very rare compared to polycystic ovarian syndrome, routine screening is not indicated in women with hypoandrogenic anovulation. Acne, hirsutism, androgenic alopecia, and acanthosis nigricans are all consistent with polycystic ovarian syndrome.

False-positive urine screens for drug abuse can occur as a result of (check one) A. passive inhalation of crack cocaine B. passive inhalation of marijuana smoke C. eating poppy seed muffins D. consuming products containing hemp E. use of black cohosh

Eating as little as one poppy seed muffin can produce amounts of morphine and codeine detectable by immunoassay, as well as by gas chromatography and mass spectrometry. Passively inhaled crack cocaine or marijuana (unless an extreme amount is inhaled), and ingested products containing hemp or other common herbal preparations do not produce positive urine drug screens. In addition to poppy seeds, substances reported to cause false-positive urine drug screens include selegiline, Vicks inhalers, NSAIDs, oxaprozin, fluoroquinolones, rifampin, venlafaxine, and dextromethorphan.

Patients often use echinacea for the prevention and treatment of: (check one) A. Memory loss B. Upper respiratory symptoms C. Gastrointestinal illnesses D. Depression E. Fatigue

Echinacea is a genus of native North American plants commonly known as purple coneflower. It has been recommended as a prophylactic treatment for upper respiratory infection, and is widely used for this indication, although it appears to be relatively ineffective. The research is difficult to evaluate because of the heterogeneity of the products used in various studies.

A 30-year-old female complains of dysmenorrhea, pelvic pain, and dyspareunia. Which one ofthe following would be appropriate to detect endometriosis? (check one) A. A CA-125 assay B. Transvaginal ultrasonography C. CT of the pelvis D. MRI of the pelvis E. Colonoscopy

Endometriosis is caused by menstrual tissue in the pelvic peritoneal cavity. Infertility, dysmenorrhea, and dyspareunia with postcoital bleeding are common. Although laparoscopy with histology is the definitive test, transvaginal ultrasonography is the noninvasive test of choice. CA-125 will often be elevated but is nonspecific. CT and MRI also have low specificity, and colonoscopy is of no value in the evaluation ofendometriosis.

Which one of the following is the recommended duration of thromboprophylaxis following total hip arthroplasty, starting from the day of surgery and including outpatient prophylaxis? (check one) A. 7 days B. 14 days C. 35 days D. 60 days E. 90 days

For patients undergoing major orthopedic surgery, the American College of Chest Physicians recommends outpatient thromboprophylaxis for a duration of up to 35 days. Older recommendations for 10-14 days of prophylaxis were based on studies performed when this was the usual hospital stay. This is still recommended as the minimum length for prophylaxis, but a longer period of time is preferred.

Which one of the following metabolic abnormalities is most likely to be seen in patients with stage 4 kidney disease? (check one) A. Hyperaldosteronism B. Hyperparathyroidism C. Hypothyroidism D. Hypogonadism E. Type 2 diabetes mellitus

Hyperparathyroidism is present in more than half of patients who have a glomerular filtration rate <60 mL/min, and is independently associated with increased mortality and an increased prevalence of cardiovascular disease. In patients with stage 4 chronic kidney disease, current guidelines recommend monitoring of serum calcium and phosphate levels every 3-6 months and bone-specific alkaline phosphatase activity every 6-12 months with the goal of normalizing these values. The other metabolic abnormalities listed are less common than hyperparathyroidism.

A 40-year-old white male was seen 4 weeks ago for a sudden onset of cough and shortness of breath. At that visit his O2 saturation was 92%, but his examination and a chest radiograph were normal. You prescribed azithromycin (Zithromax) and an albuterol inhaler (Proventil, Ventolin). Ten days later he was feeling well and his oxygen saturation was 97%. Today he returns to the office with a dry cough and shortness of breath.On examination he has rare inspiratory rales that clear with deep breaths, and he has an O2 saturation of 86%. A chest film and a D-dimer test are normal. Pulmonary function tests show significant restriction that improves only minimally with albuterol. He has not been exposed to anyone with a similar illness, has no history of asthma, and has no smoking history or occupational exposure. However, he reports that 2 months ago his home was flooded after a heavy rain, and he has been tearing out carpeting that was ruined by the flood.Which one of the following is the most likely diagnosis? (check one) A. Persistent asthma with acute exacerbations B. Legionnaires' disease C. Pulmonary embolism D. Hypersensitivity pneumonitis

Hypersensitivity pneumonitis can present in acute, subacute, or chronic forms. The case described includes two episodes of the acute form. The patient was exposed to mold antigens in his flooded home. Within 4-8 hours of exposure, chills, cough, and shortness of breath will be noted, and at times will be dramatic. A chest film can be normal, even with significant hypoxia. Pulmonary function tests will show restrictive changes, as compared to the reversible obstructive changes of acute asthma. Blood tests often show an elevated erythrocyte sedimentation rate. Serum IgG tests for the probable antigen confirm the diagnosis.Symptoms of acute hypersensitivity pneumonitis resolve over several days, but will suddenly and violently recur with repeated exposure to the offending antigen. The subacute form begins gradually over weeks or months, causing a cough and increasing shortness of breath. The chronic form develops over years of exposure, causing fibrotic changes to the lungs that will be evident on radiographs, as well as chronic crackles on auscultation. Asthma would be an unlikely diagnosis in this case, with the pulmonary function tests showing restrictive changes rather than obstructive changes, and little improvement with albuterol. Also, the lack of a previous history of asthma makes it less likely. Legionnaires' disease is always possible, but is unlikely in this case given the sudden onset, quick recovery over several days, and sudden recurrence. Pulmonary embolism is ruled out by the negative D-dimer test.

Lymphadenopathy of the head and neck at which one of the following sites is most likely to be malignant? (check one) A. Anterior cervical B. Posterior cervical C. Preauricular D. Submandibular E. Supraclavicular

In patients with head and neck lymphadenopathy, supraclavicular nodes are the most likely to be malignant. Lymphadenopathy of these nodes should always be investigated, even in children. Overall, the prevalence of malignancy with this presentation is unknown, but rates of 54%-85% have been seen in biopsy series reports.

An abandoned infant is brought to the hospital for evaluation. Based on the presence of a dried umbilical cord remnant and her overall appearance, you believe her to be no more than 5 days of age. A thorough examination is normal except for a finding of bilateral conjunctival erythema and exudate. A Gram stain of the exudate is remarkable for numerous WBCs, very few of which are noted to contain gram-negative diplococci.Which one of the following treatment options is most appropriate? (check one) A. Application of moist, warm saline eye compresses B. Irrigation of both eyes with povidone-iodine (Betadine) C. One-time application of ophthalmic erythromycin ointment into both eyes D. Instillation of silver nitrate solution into both eyes E. Intramuscular injection of ceftriaxone (Rocephin)

Infantile gonococcal infection is usually the result of exposure to infected cervical exudate during delivery and manifests 2-5 days after birth. Ophthalmia neonatorum and sepsis are the most severe gonococcal infections in newborns and immediate treatment is warranted based on the presumptive diagnosis. Topical antibiotics are appropriate for prophylaxis, but not for treatment. Silver was used for prophylaxis at one time, but is no longer available. Povidone-iodine has not been studied for prevention. A single dose of 25-50 mg/kg of ceftriaxone administered intravenously or intramuscularly is the recommended treatment.

Three members of the same family present with a high fever and cough that began abruptly yesterday. All three report having fevers over 40° C (104° F), painful coughs, moderate sore throats, and prostration. They have loss of appetite, but no vomiting or diarrhea. Two other family members have similar symptoms. On examination the patients appear ill and flushed. There is no cervical adenopathy, no visible pharyngeal inflammation, and no significant findings on examination of the chest. Which one of the following is the most likely diagnosis? (check one) A. Mycoplasma pneumonia B. Influenza-like illness C. Bacterial bronchitis D. Upper respiratory infection E. Legionnaires disease

Influenza has a very abrupt onset, and a fever with a nonproductive cough is almost always present. Unconfirmed cases are referred to as influenza-like illness (ILI) or suspected influenza. Patients with confirmed cases tend to say they have never been so ill. Mycoplasma pneumonia can spread among family members, but it is milder and has a more indolent onset and a longer incubation period. Bacterial bronchitis is an overdiagnosed, supposed complication of upper respiratory infections, and is not contagious. While the phrase cold and flu is often used, upper respiratory infections are not so febrile or prostrating, and coryza is the dominant syndrome sooner or later. Legionella can have point-source epidemics, but the incubation period is longer, symptoms vary from mild illness to life-threatening pneumonia, and diarrhea is prominent in many cases.

A 28-year-old gravida 2 para 1 at 32 weeks' gestation presents with severe itching. She denies fever or vomiting. Her physical examination is remarkable for jaundice, but is otherwise benign. Laboratory studies reveal a normal CBC, normal platelets, normal glucose and serum creatinine levels, normal transaminase levels, and a bilirubin level of 4.0 mg/dL (N 0.0-1.0). Which one of the following is the most likely diagnosis? (check one) A. Intrahepatic cholestasis of pregnancy B. Acute viral hepatitis C. Acute fatty liver of pregnancy D. Pruritic urticarial papules and plaques of pregnancy (PUPPP) E. Hemolysis, elevated liver enzymes, low platelets (HELLP) syndrome

Intrahepatic cholestasis of pregnancy is rare, occurring in 0.01% of pregnancies. It usually presents in the third trimester. Approximately 80% of patients present with pruritus alone, and another 20% with jaundice and pruritus. Laboratory results usually reveal normal or minimal elevation in transaminase levels, elevated bilirubin (usually <5 mg/dL), and occasional elevations in cholesterol and triglyceride levels. It is important to recognize and diagnose this entity, as it is associated with prematurity, fetal distress, and increased perinatal mortality. Acute viral hepatitis is a common cause of jaundice in pregnancy; however, it usually does not present with severe pruritus, and transaminase levels are markedly elevated. Acute fatty liver of pregnancy is another rare condition occurring in the third trimester and is usually associated with preeclampsia (50%-100% of cases). It presents with nausea and vomiting, anorexia, jaundice, abdominal pain, headache, and neurologic abnormalities. Transaminase levels are moderately elevated, PT and PTT are prolonged, and profound hypoglycemia and renal failure are usually present. Pruritic urticarial papules and plaques of pregnancy (PUPPP) is more common in women that present with severe pruritus. However, jaundice and liver function abnormalities are absent. HELLP syndrome is an uncommon but serious condition which presents in the third trimester with hemolysis, elevated transaminases, and low platelet count.

Which one of the following has been shown to have a beneficial effect for symptoms of the common cold in an adult? (check one) A. Diphenhydramine (Benadryl) B. Ipratropium (Atrovent) nasal spray C. Intranasal zinc D. Intranasal corticosteroids E. Systemic corticosteroids

Ipratropium is the only nasally inhaled anticholinergic recommended by the American College of Chest Physicians for a cough caused by the common cold. One study showed that the nasal formulation decreases rhinorrhea and sneezing, and a Cochrane review found that ipratropium bromide nasal spray improved rhinorrhea but did not help nasal stuffiness (SOR B). Antihistamine monotherapy (either sedating or nonsedating) such as diphenhydramine was no more effective than placebo (SOR A). Corticosteroids have not been found to be effective for the symptoms of a common cold. Intranasal zinc should not be used because it may result in the permanent loss of smell.

Which one of the following complications occurs most frequently after Roux-en-Y gastric bypass surgery for obesity? (check one) A. Early dumping syndrome B. Late dumping syndrome C. Pulmonary embolism D. Iron and vitamin B12 deficiency

Iron and vitamin B12 deficiencies develop in more than 30% of patients after Roux-en-Y gastric bypass. The incidence of pulmonary embolus is 1%-2%. The incidence of dumping syndrome is very low.

In the U.S., the most common nutritional deficiency is: (check one) A. Iron B. Vitamin B12 C. Vitamin D D. Calories E. Protein

Iron deficiency is the most common known form of nutritional deficiency. Its prevalence is highest in children and in women of childbearing age (especially pregnant women). Ref: Recommendations to prevent and control iron deficiency in the United States.

Of the following, which one is the most common adverse event to complicate the hospital course of patients age 65 and over? (check one) A. Falls B. Wound infections C. Drug-related events D. Procedure-related events E. Anesthesia-related events

It has been observed that drug-related problems are the most common type of adverse event, and for hospitalized patients the rate of these events increases with the patient's age. One study showed that in patients who are >65 years of age, the number of events per 1000 discharges was 11.46 for drug-related events, 6.15 for wound infection, 3.85 for procedure-related events, 3.19 for falls, and 0.09 for anesthesia-related events.

Patients with symptomatic heart failure associated with a reduced systolic ejection fraction or left ventricular remodeling should be initially treated with which one of the following agents? (check one) A. An ACE inhibitor B. Hydralazine (Apresoline) C. Warfarin (Coumadin) D. Amiodarone (Cordarone) E. Verapamil (Calan, Isoptin)

It has been shown that congestive heart failure (CHF) patients treated with ACE inhibitors survive longer, and all such patients should take these agents if tolerated. Warfarin and/or antiarrhythmic drugs should be given only to selected CHF patients. Verapamil may adversely affect cardiac function and should be avoided in patients with CHF. Hydralazine can be used, but because of its side effect profile would be a second-line agent.

An otherwise healthy 1-year-old white male has a screening hemoglobin level of 10.5 g/dL (N 11.3-14.1), a mean corpuscular volume of 68 fL (N 71-84), and an undetectable serum lead level. What should be your next step? (check one) A. A therapeutic trial of iron for 1 month B. A serum ferritin level C. An erythrocyte protoporphyrin level D. Hemoglobin electrophoresis E. Bone marrow examination

It is important to screen for anemia during late infancy. Iron deficiency is the most common cause of anemia in this age group. There is evidence that persistent iron deficiency in childhood may have a negative impact on cognitive development. A therapeutic trial of iron is the best approach to the treatment of iron deficiency in late infancy. If the anemia fails to respond, investigating other causes of anemia is indicated.

You see a 16-year-old white female for a preparticipation evaluation for volleyball. She is 183 cm (72 in) tall, and her arm span is greater than her height. She wears contacts for myopia. Which one of the following should be performed at this time? (check one) A. An EKG B. Echocardiography C. A stress test D. A chest radiograph E. Coronary MRI angiography

Marfan's syndrome is an autosomal dominant disease manifested by skeletal, ophthalmologic, and cardiovascular abnormalities. Men taller than 72 in and women taller than 70 in who have two or more manifestations of Marfans disease should be screened by echocardiography for associated cardiac abnormalities. Any of these athletes who have a family history of Marfan's syndrome should be screened, whether they have manifestations themselves or not. If there is no family history, echocardiography should be performed if two or more of the following are present: cardiac murmurs or clicks, kyphoscoliosis, anterior thoracic deformity, arm span greater than height, upper to lower body ratio more than 1 standard deviation below the mean, myopia, or an ectopic lens. Patients with Marfan's syndrome who have echocardiographic evidence of aortic abnormalities should be placed on beta-blockers and monitored with echocardiography every 6 months.

A 14-year-old male is brought to your office by his mother to establish care. The patient has been diagnosed with asthma, but has not been on any medications for the past year. When questioned, he reports that his asthmatic symptoms occur daily and more than one night per week. On examination, he is found to have a peak expiratory flow of 75%. Based on these findings, the most accurate classification of this patient's asthma is: (check one) A. Mild intermittent B. Mild persistent C. Moderate persistent D. Severe persistent

Moderate persistent The National Asthma Education and Prevention Program (NAEPP) classifies asthma into four categories. Mild intermittent asthma is characterized by daytime symptoms occurring no more than 2 days per week and nighttime symptoms no more than 2 nights per month. The peak expiratory flow (PEF) or forced expiratory volume in 1 second (FEV1) is 80% or more of predicted. Mild persistent asthma is characterized by daytime symptoms more than 2 days per week, but less than once a day, and nighttime symptoms more than 2 nights per month. PEF or FEV1 is 80% or more of predicted. Moderate persistent asthma is characterized by daytime symptoms daily and nighttime symptoms more than 1 night per week. PEF or FEV1 is 60%-80% of predicted. Severe persistent asthma is characterized by continuous daytime symptoms and frequent nighttime symptoms. PEF or FEV1 is 60% or less of predicted.

Which one of the following is a classic finding in multiple myeloma? (check one) A. Hypokalemia B. Bone pain C. Polycythemia D. Hepatic failure E. Insomnia

Multiple myeloma can be asymptomatic, but it becomes symptomatic when there is organ damage or other abnormalities, including renal insufficiency, elevated calcium, anemia, and bone disease.The majority of patients have bone pain, but hypocalcemia is not common. Hypokalemia almost never occurs, and both hepatic failure and insomnia are not usual signs of multiple myeloma. Anemia typically occurs either because of renal failure or infiltration of the bone marrow by myeloma cells. Polycythemia does not occur.

A 7-year-old African-American male is brought to your office with a 1-day history of purulent, crusted eyelashes in the morning, and red eye. There is no history of visual change, foreign body, or injury. The child is otherwise in good health and has normal developmental milestones. No fever or respiratory distress is noted. A clinical diagnosis of bacterial conjunctivitis is made. The mother is anxious to keep the child in school. Which one of the following would be the most appropriate time for the child to return to school? (check one) A. Once treatment is started B. When there is no crusting or drainage in the morning C. After 1 week of treatment D. When the absence of fever for 24 hours is documented

Once therapy is initiated, children with bacterial conjunctivitis should be allowed to remain in school. Careful hand hygiene is important, however, and behavior must be appropriate to maintain adequate hygiene. No specific length of treatment or evidence of clinical response is required before returning to school.

A 68-year-old female presents with recent poor oral intake, fatigue, and confusion. Osmotic demyelination syndrome (central pontine myelinolysis) and permanent neurologic deficits could result from overly rapid correction of which one of the following abnormalities? (check one) A. Hyperglycemia B. Hyperkalemia C. Hypokalemia D. Hypernatremia E. Hyponatremia

Overly rapid correction of hyponatremia may cause osmotic demyelination syndrome, or central pontine myelinolysis, sometimes resulting in permanent neurologic deficits after a brief improvement in neurologic status. Signs and symptoms may include dysarthria, dysphagia, paresis, coma, and seizures. It is believed that brain volume shrinks because it cannot assimilate the new electrolytes fast enough and water is lost from the cells. Rapid correction of hypernatremia that has been present for a short time is relatively safe. Hyperkalemia is a life-threatening condition that should be corrected promptly. Rapid correction of hypoglycemia is not an issue. Overly rapid correction of hyperglycemia and subsequent cerebral edema is unusual and is primarily seen in children.

A 51-year-old female has resistant hypertension, and you decide to test her for primaryhyperaldosteronism. Which one of the following is the preferred initial test for this condition? (check one) A. A morning serum cortisol level B. A morning serum renin to aldosterone ratio C. A morning urinary potassium level D. A salt suppression test E. Abnominal MRI

Primary hyperaldosteronism is a relatively common cause of resistant hypertension. Because there are effective treatments, it is reasonable to consider testing for hyperaldosteronism in patients with resistant hypertension. This is true even for patients with a normal potassium level. The preferred initial test is a morning renin to aldosterone ratio. A ratio <20 (when plasma aldosterone is reported in ng/dL and plasmarenin activity is in ng/mL/hr) effectively rules out primary hyperaldosteronism. A ratio ³20 with a serum aldosterone level >15 ng/dL suggests aldosteronism, but a salt suppression test must be done for confirmation. Although abdominal MRI may detect an adrenal mass, it is not recommended as a test forhyperaldosteronism. Urinary potassium levels do not play a role in the diagnosis of primaryhyperaldosteronism.

Which one of the laboratory abnormalities is an indication that he has severe diabeticketoacidosis? (check one) A. Glucose B. Sodium C. Potassium D. Bicarbonate E. BUN

The diagnosis of diabetic ketoacidosis (DKA) is based on an elevated serum glucose level (>250 mg/dL),an elevated serum ketone level, a pH <7.3, and a serum bicarbonate level <18 mEq/L. The severity ofDKA is determined by the arterial pH, bicarbonate level, anion gap, and mental status of the patient.Elevation of BUN and serum creatinine levels reflects intravascular volume loss. The measured serumsodium is reduced as a result of the hyperglycemia, as serum sodium is reduced by 1.6 mEq/L for each100 mg/dL rise in serum glucose. The degree of hyperglycemia does not necessarily correlate closely withthe degree of DKA since a variety of factors determine the level of hyperglycemia, such as oral intake andurinary glucose loss (SOR C).

You are evaluating a 5-month-old with fever, tachypnea, and mild respiratory distress in the emergency department. You hear mild basilar rales. The child does not appear toxic. Which one of the following tests would be the most appropriate as an initial study? (check one) A. A chest radiograph B. A CBC C. A C-reactive protein level D. Oxygen saturation by pulse oximetry

Pulse oximetry should be obtained on all pediatric patients with significant tachypnea, pallor, or respiratory distress. It has been found that CBCs, C-reactive protein levels, and erythrocyte sedimentation rates are not effective in differentiating between viral and bacterial pneumonia. Chest radiographs are also ineffective in distinguishing viral and bacterial pneumonia, and should be obtained in cases of ambiguous clinical findings, prolonged pneumonia, and pneumonia that is unresponsive to antibiotic therapy, as well as when there is the possibility of complications such as pleural effusions.

A 4-week-old full-term male is brought to your office by his parents. They report that their child started vomiting just after his 1-week visit. The parents are concerned because they think the vomiting is worsening, occurring after every feeding, and "shooting across the room." You note that the baby is afebrile, but has not gained any weight since birth. Based on this information, the most likely diagnosis is: (check one) A. Formula intolerance B. Meningitis C. Viral gastroenteritis D. Pyloric stenosis E. Inappropriate feeding

Pyloric stenosis fits the described scenario; it is characterized by the early onset of worsening projectile vomiting and poor weight gain, and occurs most often in full-term male infants who are otherwise healthy. Formula intolerance causes regurgitation, as would inappropriate feeding. Meningitis, whether viral or bacterial, would be associated with fever. Viral gastroenteritis is a common cause of vomiting in older children, and is usually associated with fever and diarrhea.

A 13-year-old male presents with a 3-week history of left lower thigh and knee pain. There isno history of a specific injury, and his past medical history is negative. He has had no fevers,night sweats, or weight loss, and the pain does not awaken him at night. He tried out for hisschool's basketball team but had to quit because of the pain, which was worse when he tried torun.Which one of the following physical examination findings would be pathognomonic for slippedcapital femoral epiphysis? (check one) A. Excessive forward passive motion of the tibia with the knee flexed B. Lateral displacement of the patella with active knee flexion C. Limited internal rotation of the flexed hip D. Reduced hip abduction with the hip flexed E. An inability to extend the hip past the neutral position

Slipped capital femoral epiphysis (SCFE) typically occurs in young adolescents during the growth spurt,when the femoral head is displaced posteriorly through the growth plate. Physical activity, obesity, and male sex are predisposing factors for the development of this condition. There is pain with physicalactivity, most commonly in the upper thigh anteriorly, but one-third of patients present with referred lowerthigh or knee pain, which can make accurate and timely diagnosis more difficult.The hallmark of SCFE on examination is limited internal rotation of the hip. Specific to SCFE is the even greater limitation of internal rotation when the hip is flexed to 90°. No other pediatric condition has thisphysical finding, which makes the maneuver very useful in children with lower extremity pain. Orthopedic consultation is advised if SCFE is suspected.Hip extension and abduction are also limited in SCFE, but these findings are nonspecific. Displacement of the patella is not associated with SCFE.

A 78-year-old white female presents with a 3-day history of lower thoracic back pain. She denies any antecedent fall or trauma, and first noted the pain upon arising. Her description of the pain indicates that it is severe, bilateral, and without radiation to the arms or legs. Her past medical history is positive for hypertension and controlled diabetes mellitus. Her medications include hydrochlorothiazide, enalapril (Vasotec), metformin (Glucophage), and a general multivitamin. She is a previous smoker but does not drink alcohol. She underwent menopause at age 50 and took estrogen for "a few months" for hot flashes. Physical examination reveals her to be in moderate pain with a somewhat stooped posture and mild tenderness over T12-L1. She has negative straight-leg raising and normal lower extremity sensation, strength, and reflexes. Which one of the following is true regarding this patient's likely condition? (check one) A. An MRI or nuclear medicine bone scan should be performed B. Prolonged (approximately 2 weeks) bed rest will increase the chance of complete recovery C. Investigation for an underlying malignancy is indicated D. Subcutaneous or intranasal calcitonin (Calcimar, Miacalcin) may be very helpful for pain relief

Subcutaneous or intranasal calcitonin (Calcimar, Miacalcin) may be very helpful for pain relief The patient described has a classic presentation of an osteoporotic vertebral compression fracture. The diagnosis should be confirmed with a plain radiograph. Treatment is basically symptomatic, with a period of bed rest as short as possible (to avoid complications of immobility), pain medication, and bracing. Salmon calcitonin (injectable or intranasal) is often helpful in providing pain relief. Long-term management of underlying osteoporosis may help prevent future fractures.

You are considering recommending surgical treatment for obesity in selected patients. All other attempts to control weight have failed in these patients, including diet education, medication, exercise, and behavior modification. Each of these individuals is a well-informed and motivated patient with acceptable operative risks and is able to participate in treatment and long-term follow-up. They strongly desire substantial weight loss because their obesity impairs the quality of their lives, and they have asked about surgical options. Which one of these patients would meet the criteria for surgical treatment of obesity? (check one) A. A 44-year-old with a BMI of 34 and degenerative joint disease of the knees that significantly limits his ability to walk B. A 45-year-old with a BMI of 36 and controlled diabetes mellitus C. A 48-year-old with a BMI of 42 and no other health problems D. A 52-year-old with a BMI of 29 and sleep apnea E. A 55-year-old with a BMI of 29 and uncontrolled diabetes mellitus

The 1991 National Institutes of Health Consensus Development Panel recommended that surgical treatment of severe obesity be considered for any patient with a BMI >40 or those with a BMI >35 who have serious coexisting medical problems. Examples of such coexisting medical problems include severe sleep apnea, Pickwickian syndrome, obesity-related cardiomyopathy, and severe diabetes mellitus.

Which one of the following is most likely to induce withdrawal symptoms if discontinued abruptly? (check one) A. Venlafaxine (Effexor) B. Divalproex (Depakote) C. Fluoxetine (Prozac) D. Olanzapine (Zyprexa) E. Donepezil (Aricept)

The abrupt discontinuation of venlafaxine, or a reduction in dosage, is associated with withdrawal symptoms much more severe than those seen with other SSRIs such as fluoxetine. Although more pronounced with higher dosages and prolonged administration, they also occur at lower dosages. These symptoms include agitation, anorexia, confusion, impaired coordination, seizures, sweating, tremor, and vomiting. To avoid this withdrawal symptom, dosage changes should be instituted gradually. Abrupt discontinuation of mood stabilizers such as divalproex, and atypical antipsychotics such as olanzapine, can result in the return of psychiatric symptoms, but not severe physiologic dysfunction. Similarly, stopping anticholinesterase inhibitors such as donepezil will not cause a withdrawal syndrome.

You are considering how useful a new treatment might be in preventing stroke. A well designed study is reported with 200 patients in the treated group and 200 patients in the untreated group. The study finds a 5-year risk of stroke of 3% in the treated group versus 5% in the untreated group. Assuming this study is valid and applicable to your patient population, how many patients would you have to treat for 5 years to prevent one stroke (number needed to treat, or NNT)? (check one) A. 400 B. 200 C. 100 D. 50 E. 25

The relative risk reduction (RRR) is the proportional decrease in disease incidence in the treated group relative to the incidence in the control group. In this example the 3% incidence in the treated group is 40% less than the 5% incidence in the control group: (5%-3%)/5% = 40%. The absolute risk reduction (ARR) is the difference between the incidence of disease in the treatment group and the incidence in the control group. In this example the ARR is 5% minus 3% = 2%. The number needed to treat (NNT) equals the reciprocal of the ARR: 1/.02 = 50. The RRR is not a very useful statistic in clinical practice. It amplifies small differences and makes clinically insignificant findings appear significant because it essentially ignores the baseline risk of the outcome event. The ARR provides a more useful measure of clinical effect. It answers the question "How much will I decrease my patient's risk of an adverse outcome by this treatment?" The NNT is also very useful for clinicians, as it answers the question, "How many patients will I need to treat to prevent one adverse outcome?"

A new home test is designed to detect a particular type of cancer. The gold standard test for this cancer is a biopsy, but a biopsy is more costly that a urine test, is invasive, and is associted with a number of adverse side effects. To test the effectiveness of the home urine test, 104 people took the test and then agreed to a biopsy. When the study was concluded, 77 people tested negative and 27 tested positive on the urine test. Biopsies were positive in 18 individuals, 8 of whom tested negative on the urine test.What is the negative predictive value of the home urine test, rounded to a whole number? (check one) A. 20% B. 37% C. 56% D. 80% E. 90%

The results of this urine test were that 10 people had the disease and tested positive (true positives); 8 people had the disease but tested negative (false negatives); 17 people did not have the disease (27 - 10) but tested positive (false positive); 69 people did not have the disease (77 - 8) and tested negative (true negative). The negative predictive value is determined by dividing the true negatives (69) by the total number who tested negative (true plus false negatives = 77). The result is 89.6%, which rounded to a whole number is 90%.

The scabies mite is predominantly transmitted by: (check one) A. Bedclothes B. Personal contact C. Hats D. Pets

The scabies mite is predominantly transmitted by direct personal contact. Infestation from indirect contact with clothing or bedding is believed to be infrequent. Hats are frequent transmitters of head lice, but not scabies.

For most patients, which one of the following is the most effective treatment for anemia of chronic disease? (check one) A. Elemental iron B. Erythropoietin C. Prednisone D. Optimal management of the underlying disorder E. Combined therapy with oral iron, vitamin B12, folic acid, and erythropoietin 0.3%/dexamethasone 0.1% (Ciprodex) topically

There is no specific therapy for anemia of chronic disease except to manage or treat the underlying disorder. Iron therapy is of no benefit, but erythropoietin may be helpful in some patients. There is no available data to suggest that combination therapy or prednisone is beneficial for this disorder.

A 35-year-old white female comes to your office with a 3-month history of the gradual onset of pain and tenderness in her wrists and hands. She also complains of 1 hour of morning stiffness. She denies rash, fever, or skin changes. On physical examination she has symmetric swelling of the proximal interphalangeal joints and metacarpophalangeal joints. Motion of these joints is painful. She has no rash or mouth ulcers. Radiographs of the hands and wrists are negative, and a chest film is unremarkable. A CBC is normal, but the erythrocyte sedimentation rate is elevated at 40 mm/hr. Latex fixation for rheumatoid factor is negative, and an antinuclear antibody (ANA) test is negative.The most likely diagnosis in this patient is (check one) A. rheumatoid arthritis B. systemic lupus erythematosus C. sarcoidosis D. Lyme disease

This patient has rheumatoid arthritis (RA) by symptoms and physical findings. A positive latex fixation test for rheumatoid factor is not necessary for the diagnosis. A negative rheumatoid factor does not exclude RA, and a positive rheumatoid factor is not specific. Rheumatoid factor is found in the serum of approximately 85% of adult patients with RA; in subjects without RA, the incidence of positive rheumatoid factor is 1%-5% and increases with age.The ANA test is positive in at least 95% of patients with systemic lupus erythematosus, but in only about 35% of patients with RA. Elevation of the erythrocyte sedimentation rate is seen in many patients with RA, and the degree of elevation roughly parallels disease activity. At a mean of 6 months after the onset of Lyme disease, 60% of patients in the United States have brief attacks of asymmetric, oligoarticular arthritis, primarily in the large joints and especially in the knee.

An 85-year-old male is admitted to a nursing home due to weakness, debility, and limitation of activities of daily living (ADLs) after being hospitalized for acute community-acquired pneumonia. He previously lived with his wife independently and his goal is to return home when he is strong enough. He has a history of coronary artery disease, type 2 diabetes mellitus controlled with diet, hypertension, and chronic diastolic heart failure, but he has no symptoms related to these chronic problems. His appetite is poor and he has lost a significant amount of weight. His admission diet order from the hospital was a cardiac diet.Which one of the following would be the most appropriate diet for this patient? A. A regular diet B. An American Heart Association diet C. A diet with no added salt D. An 1800-calorie/day American Diabetes Association diet E. A diet with no concentrated sweets

This patient should be provided with a regular diet, which may promote weight gain in nursing-home residents with unintentional weight loss. Malnutrition and unintentional weight loss are significant problems in nursing-home residents and lead to multiple complications, including pressure ulcers and infections. The American Dietetic Association recommends liberalizing diets to improve nutritional status and quality of life in older adults. A small study demonstrated equivalent glycemic control in nursing-home residents who ate a regular diet compared to those who ate a restricted American Diabetes Association diet (SOR C). Low-salt and low-cholesterol diets are unpalatable and are often associated with protein-energy malnutrition and postural hypotension in older persons. Special diets should be avoided whenever possible in nursing-home patients.

You receive a telephone call from the mother of a 5-year-old female. The child has had diarrhea and a decreased appetite for the past 2 days. She is still playing some. The mother reports no vomiting, but says her daughter has complained of a dry mouth and does not have tears when she cries. You suspect that the child may be mildly dehydrated.Which one of the following would you advise? (check one) A. Increased water intake B. Clear liquids with sodium, such as chicken broth C. An over-the-counter oral rehydration solution D. Intravenous fluids in the emergency department E. Loperamide (Imodium)

When children show signs of dehydration from diarrhea, the first step is to assess its extent. In one study, four factors predicted dehydration: a capillary refill time >2 seconds, the absence of tears, dry mucous membranes, and an ill general appearance; the presence of two or more of these signs indicates a fluid deficit of at least 5%. This child has two of the signs, but does not require intravenous fluids at this point. Early oral rehydration therapy is recommended and can be started at home. This should be done using an oral rehydration solution that is designed for children (SOR C). Adult oral rehydration solutions should not be used in children.Water and other clear liquids, even those with sodium, such as chicken broth, should not replace an oral rehydration solution because they are hyperosmolar. These fluids do not adequately replace potassium, bicarbonate, or sodium, and can sometimes cause hyponatremia. Antidiarrheal medications are usually not recommended for use in children with acute gastroenteritis because they delay the elimination of infectious agents from the intestines.


Ensembles d'études connexes

*Surface Area & Volume of Rectangular Prisms

View Set

THYROID GLAND 1 (GENERAL) - BLOOD SUPPLY

View Set

Exploring Psychology (Chapter 14)

View Set